Soluções Astronomia - Semana 82

INICIANTE

a) Para calcular a escala de placa devemos lembrar da fórmula:

p=\dfrac{1}{f}

Segundo o desenho a garrafa possui 30cm de comprimento, mas, como o resultado deve ser dado em rad/mm, utilizaremos 300mm nos cálculos.

p = \dfrac{1}{300}\rightarrow \boxed{p = 3,\overline{3} \cdot 10^{-3} \dfrac{rad}{mm} }

b)O campo de visão de um telescópio utilizado para observação visual é definido pela seguinte relação:

FOV=\dfrac{AFOV}{A}

Já o aumento (A) pode ser calculado utilizando:

A=\dfrac{f_{tel}}{f_{ocu}}

Como o telescópio possui 300mm de distância focal e a ocular 25mm:

A=\dfrac{300}{25}\rightarrow A=12x

Como encontramos o A e o enunciado nos deu o AFOV:

FOV=\dfrac{70^{\circ}}{12}\rightarrow{}\boxed{FOV=5^{\circ}50'}

INTERMEDIÁRIO

Já que m<<M podemos escrever a terceira lei de Kepler como:

p^2=a^3\cdot\dfrac{4\pi^2}{GM}

\therefore p=\sqrt{\dfrac{a^34\pi^2}{GM}}

Já que:

f_{orb}=\dfrac{1}{p}\rightarrow{}\dfrac{1}{2\pi}\sqrt{\dfrac{GM}{a^3}}

f_{grav}=2f_{orb}\rightarrow{}\dfrac{1}{\pi}\sqrt{\dfrac{GM}{a^3}}

Como a questão pede para utilizar \dfrac{M}{M_\odot}:

\dfrac{R}{R_\odot}=\left(\dfrac{M}{M_\odot}\right)^\alpha \rightarrow R=R_\odot \left(\dfrac{M}{M_\odot}\right)^\alpha

Dado que a frequência será máxima quando a=R:

f_{grav}=\dfrac{1}{\pi}\sqrt{\dfrac{GM}{R^3}}

\therefore f_{grav}=\dfrac{1}{\pi}\sqrt{\dfrac{GM}{R_\odot^3}} \left(\dfrac{M_\odot}{M}\right)^\dfrac{3\alpha}{2}

f_{grav}=\dfrac{1}{\pi}\sqrt{\dfrac{GM_\odot}{R_\odot^3}} \left(\dfrac{M_\odot}{M}\right)^\dfrac{(3\alpha - 1)}{2}

\boxed{f_{grav}=\dfrac{1}{\pi}\sqrt{\dfrac{GM_\odot}{R_\odot^3}} \left(\dfrac{M}{M_\odot}\right)^\dfrac{(1-3\alpha)}{2}}

AVANÇADO

a) Primeiramente, calcula-se o valor de H_0 com base na idade do universo:

t_u=\dfrac{1}{H_0} \rightarrow H_0=\dfrac{1}{t_u}

Transformando t_u em segundos e aplicando na fórmula:

H_0\approx2,3\cdot 10^{-18} \cdot \dfrac{1}{s}

Curiosamente, esse é exatamente o resultado que precisamos, indica que em uma região de comprimento y, o espaço expandiu 2,3\cdot 10^{-18}\cdot y em 1 segundo, o parâmetro x que procuramos. Você poderia realizar a transformação para unidades mais usuais e realizar o caminho inverso, mas chegaria no mesmo resultado, como demonstrado abaixo:

Alterando a unidade para o usual:

H_0=2,3\cdot 10^{-18} \cdot \dfrac{1000000 \cdot 206265 \cdot 1.496 \cdot 10^{11}}{1000}

H_0\approx70,9 \cdot \dfrac{km}{s \cdot Mpc}

Ou seja, a cada segundo, um espaço de 1 \ Mpc expande 70,9 \ km, aplicando esses dados para encontrar x:

x=\dfrac{70,9km}{1 Mpc} \rightarrow{} \dfrac{70,9\cdot1000m}{1000000\cdot206265\cdot1,496\cdot10^{11}m}

x\approx2,3\cdot 10^{-18}

Aplicando a fórmula cedida no problema:

d=\dfrac{c(e^{xt}-1)}{x} \rightarrow{} d\approx5,17\cdot10^{25}m

Transformando em anos-luz:

\boxed{d \approx 5,46\cdot10^{9} \ anos-luz}

b)Manipulando a equação cedida:

d=\dfrac{c(e^{x\cdot t}-1)}{x}

\dfrac{d\cdot x}{c} + 1 = e^{x\cdot t}

\ln \left( \dfrac{d\cdot x}{c} + 1\right) = x\cdot t

t = \dfrac {\ln \left( \dfrac{d\cdot x}{c} + 1\right)}{x}

Transformando anos-luz em metros:

d = (21 \cdot 10^{9}) \cdot (365,25 \cdot 24 \cdot 3600) \cdot c

Utilizando o valor de x encontrado no item a):

t = \dfrac{\ln{\left(\dfrac{d\cdot x}{c} + 1\right)}}{x} \rightarrow{} \dfrac{\ln{\left(\dfrac{(21 \cdot 10^{9}) \cdot (365,25 \cdot 24 \cdot 3600) \cdot c \cdot 2,3\cdot10^{-18}}{c} + 1\right)}}{2,3\cdot10^{-18}}

t=4,03\cdot10^{17} \ s \rightarrow \boxed{12,76\cdot 10^{9} \ anos}

c)Neste item o primeiro passo a se seguir é calcular o maior "raio" possível no universo:

d=\dfrac{c(e^{xt}-1)}{x}

Utilizando o H_0 encontrado x \approx 2,3 \cdot 10^{-18} e t \approx 13,8 \cdot 10^9 \ anos, chegamos em:

d \approx 2.25 \cdot 10^{26} metros.

Tendo tal raio podemos calcular o "volume total" deste universo simplificado (\approx esférico):

V_e=\dfrac{4}{3}\pi r^3

Realizando as substituições encontramos que V_e \approx 4,74 \cdot 10^{79} \ m^3

Tendo esse valor basta utilizar a equação da densidade crítica de Friedmann e calcular a massa com base no volume encontrado:

\rho_c = \dfrac{3H_0^2}{8\pi G}

\therefore \rho_c \approx 9.5 \cdot 10^{-27} \cdot \dfrac{Kg}{m^3}

Como V\cdot \rho_c = M \rightarrow M \approx 4,5 \cdot 10^{53} Kg

(Note que o valor é diferente do real já que H_0 foi considerado constante, causando um erro na medida do volume total.)